K
Khách

Hãy nhập câu hỏi của bạn vào đây, nếu là tài khoản VIP, bạn sẽ được ưu tiên trả lời.

12 tháng 9 2021

Dễ chứng minh được \(a^2+b^2+c^2\ge\frac{\left(a+b+c\right)^2}{3}\)\(\)

\(\Leftrightarrow\left(a-b\right)^2+\left(b-c\right)^2+\left(c-a\right)^2\ge0\left(true\right)\)

\(\Rightarrow2\left(a+b+c\right)\ge\frac{\left(a+b+c\right)^2}{3}\)

\(\Leftrightarrow a+b+c\le6\)

Ta có : \(T=\frac{a}{a+1}+\frac{b}{b+1}+\frac{c}{c+1}\)

\(=1-\frac{1}{a+1}+1-\frac{1}{b+1}+1-\frac{1}{c+1}\)

\(=3-\left(\frac{1}{a+1}+\frac{1}{b+1}+\frac{1}{c+1}\right)\)

\(\le3-\frac{9}{a+b+c+3}\le3-\frac{9}{6+3}=2\)

Dấu "=" xảy ra khi \(a=b=c=2\)

12 tháng 9 2021

bạn ơi , kết quả thì đúng r nhưng tại sao đoạn \(2\left(a+b+c\right)\ge\frac{\left(a+b+c\right)^2}{3}\Rightarrow a+b+c\le6\)

1 tháng 5 2018

bn sử dụng bất đẳng thức cô si đi

1 tháng 5 2018

Nguyễn Đại Nghĩa,bác nói cụ thể hơn được ko :v

26 tháng 10 2016

sao toàn toán lớp 9 thế

26 tháng 10 2016

\(a-\frac{ab^2}{b^2+1}\ge a-\frac{ab^2}{2b}=a-\frac{ab}{2}\)

Tương tự và cộng lại, ta có:\(p\ge a+b+c-\frac{ab+bc+ca}{2}\) mà 3(ab+bc+ca)\(\le\)(a+b+c)^2=9

=>ab+bc+ca\(\le\)3

=> \(p\ge3-\frac{3}{2}=\frac{3}{2}\)

Dấu = xảy ra =>a=b=c=1

25 tháng 10 2020

Bài 4: Áp dụng bất đẳng thức AM - GM, ta có: \(P=\text{​​}\Sigma_{cyc}a\sqrt{b^3+1}=\Sigma_{cyc}a\sqrt{\left(b+1\right)\left(b^2-b+1\right)}\le\Sigma_{cyc}a.\frac{\left(b+1\right)+\left(b^2-b+1\right)}{2}=\Sigma_{cyc}\frac{ab^2+2a}{2}=\frac{1}{2}\left(ab^2+bc^2+ca^2\right)+3\)Giả sử b là số nằm giữa a và c thì \(\left(b-a\right)\left(b-c\right)\le0\Rightarrow b^2+ac\le ab+bc\)\(\Leftrightarrow ab^2+bc^2+ca^2\le a^2b+abc+bc^2\le a^2b+2abc+bc^2=b\left(a+c\right)^2=b\left(3-b\right)^2\)

Ta sẽ chứng minh: \(b\left(3-b\right)^2\le4\)(*)

Thật vậy: (*)\(\Leftrightarrow\left(b-4\right)\left(b-1\right)^2\le0\)(đúng với mọi \(b\in[0;3]\))

Từ đó suy ra \(\frac{1}{2}\left(ab^2+bc^2+ca^2\right)+3\le\frac{1}{2}.4+3=5\)

Đẳng thức xảy ra khi a = 2; b = 1; c = 0 và các hoán vị

26 tháng 10 2020

Bài 1: Đặt \(a=xc,b=yc\left(x,y>0\right)\)thì điều kiện giả thiết trở thành \(\left(x+1\right)\left(y+1\right)=4\)

Khi đó  \(P=\frac{x}{y+3}+\frac{y}{x+3}+\frac{xy}{x+y}=\frac{x^2+y^2+3\left(x+y\right)}{xy+3\left(x+y\right)+9}+\frac{xy}{x+y}\)\(=\frac{\left(x+y\right)^2+3\left(x+y\right)-2xy}{xy+3\left(x+y\right)+9}+\frac{xy}{x+y}\)

Có: \(\left(x+1\right)\left(y+1\right)=4\Rightarrow xy=3-\left(x+y\right)\)

Đặt \(t=x+y\left(0< t< 3\right)\Rightarrow xy=3-t\le\frac{\left(x+y\right)^2}{4}=\frac{t^2}{4}\Rightarrow t\ge2\)(do t > 0)

Lúc đó \(P=\frac{t^2+3t-2\left(3-t\right)}{3-t+3t+9}+\frac{3-t}{t}=\frac{t}{2}+\frac{3}{t}-\frac{3}{2}\ge2\sqrt{\frac{t}{2}.\frac{3}{t}}-\frac{3}{2}=\sqrt{6}-\frac{3}{2}\)với \(2\le t< 3\)

Vậy \(MinP=\sqrt{6}-\frac{3}{2}\)đạt được khi \(t=\sqrt{6}\)hay (x; y) là nghiệm của hệ \(\hept{\begin{cases}x+y=\sqrt{6}\\xy=3-\sqrt{6}\end{cases}}\)

Ta lại có \(P=\frac{t^2-3t+6}{2t}=\frac{\left(t-2\right)\left(t-3\right)}{2t}+1\le1\)(do \(2\le t< 3\))

Vậy \(MaxP=1\)đạt được khi t = 2 hay x = y = 1

20 tháng 7 2017

thỏa cái j sửa đi

24 tháng 11 2017

fkfkbang14

11 tháng 10 2017

Ta có:

\(P=\frac{1}{\sqrt{a^2-ab+b^2}}+\frac{1}{\sqrt{b^2-bc+c^2}}+\frac{1}{\sqrt{c^2-ca+a^2}}\)

\(=\frac{1}{\sqrt{\frac{1}{4}\left(a+b\right)^2+\frac{3}{4}\left(a-b\right)^2}}+\frac{1}{\sqrt{\frac{1}{4}\left(b+c\right)^2+\frac{3}{4}\left(b-c\right)^2}}+\frac{1}{\sqrt{\frac{1}{4}\left(c+a\right)^2+\frac{3}{4}\left(c-a\right)^2}}\)

\(\le2\left(\frac{1}{a+b}+\frac{1}{b+c}+\frac{1}{c+a}\right)\)

\(\le2.\frac{1}{4}.\left(\frac{1}{a}+\frac{1}{b}+\frac{1}{b}+\frac{1}{c}+\frac{1}{c}+\frac{1}{a}\right)\)

\(=\frac{1}{a}+\frac{1}{b}+\frac{1}{c}=3\)

11 tháng 11 2017

Ta có:

\(\frac{a+1}{1+b^2}=a+1-\frac{\left(a+1\right)b^2}{1+b^2}\ge a+1-\frac{\left(a+1\right)b^2}{2b}=a+1-\frac{ab+b}{2}\left(1\right)\)

Tương tụ ta có:

\(\hept{\begin{cases}\frac{\left(b+1\right)}{1+c^2}\ge b+1-\frac{bc+c}{2}\left(2\right)\\\frac{\left(c+1\right)}{1+a^2}\ge c+1-\frac{ca+a}{2}\left(3\right)\end{cases}}\)

Từ (1), (2), (3) ta có:

\(M\ge a+b+c+3-\frac{ab+bc+ca+a+b+c}{2}\)

\(=3+3-\frac{ab+bc+ca+3}{2}\)

\(\ge\frac{9}{2}-\frac{\left(a+b+c\right)^2}{6}=3\)